Year-end appeal: Please make a donation to the OEIS Foundation to support ongoing development and maintenance of the OEIS. We are now in our 61st year, we have over 378,000 sequences, and we’ve reached 11,000 citations (which often say “discovered thanks to the OEIS”).
%I #44 Aug 16 2024 21:10:38
%S 1,2,4,6,3,10,12,7,16,18,11,22,13,5,28,30,19,20,36,23,40,42,9,46,29,
%T 31,52,37,35,58,60,15,44,66,43,70,72,25,54,78,8,82,61,55,88,65,59,68,
%U 96,27,100,102,38,106,108,71,112,85,33,89,94,79,48,126,83,130
%N Let P(n) of a sequence s(1),s(2),s(3),... be obtained by leaving s(1),...,s(n) fixed and reverse-cyclically permuting every n consecutive terms thereafter; apply P(2) to 1,2,3,... to get PS(2), then apply P(3) to PS(2) to get PS(3), then apply P(4) to PS(3), etc. The limit of PS(n) is A057063.
%C It appears that this is a permutation of the integers. - _Michel Marcus_, Feb 19 2016
%C The fact that this is a permutation is proved at the MathOverflow link below. Also from that link: a(n)+1 is prime if and only if a(n) = 2*(n-1). - _Ilya I. Bogdanov_, Feb 15 2022
%C From _Jianing Song_, Sep 27 2023: (Start)
%C Let {b(n)} be the inverse permutation of this sequence, then each number n >= 3 is moved for b(n)-2 times during the process.
%C Proof: suppose that this number is k, which is well-defined since PS(1), PS(2), ... has a limit. Suppose that PS(i+1)(c_i) = n for each i >= 0, that is, c_i is the index of n after i steps. In the (i+1)-th step, each group of PS(i+1) contains i+2 elements, and every element is moved if and only if it has index at least i+3. We obtain that k = #{i >= 0 : c_i >= i+3}.
%C Note that if c_i <= i+2 for some i, then from the (i+1)-th step on, each group contains at least i+2 numbers so the first i+2 numbers in the sequence remain fixed, which means that n = PS(i+1)(c_i) = PS(i+2)(c_i) = ... = a(c_i), so c_i = c_{i+1} = ... = b(n). This shows that {i >= 0 : c_i >= i+3} = {0, 1, ..., k-1}, which implies that k is the smallest number such that c_k <= k+2. On one hand, since c_k <= k+2, we have c_k = b(n), so b(n) <= k+2. On the other hand, from the (b(n)-1)-th step on, each group contains at least b(n) numbers so the first b(n) numbers in the sequence remain fixed, which means that PS(b(n)-1)(b(n)) = PS(b(n))(b(n)) = ... = a(b(n)) = n, so c_{b(n)-2} = b(n), and k <= b(n)-2. In conclusion, we have k = b(n)-2.
%C By the MathOverflow link, we have a(n) <= 2*n-2 for all n, where the equality holds if and only if a(n)+1 is prime. On the other hand, it is hard to get a lower bound for {a(n)}, so it is infeasible to calculate the inverse permutation of this sequence. (End)
%H MathOverflow, <a href="https://mathoverflow.net/q/416089/17581">Prime numbers from permutation</a>, 2022.
%F Conjecture: a(n) = A057033(n-1) + 1 for n > 1 with a(1) = 1. - _Mikhail Kurkov_, Mar 10 2022
%e PS(2) begins with 1,2,4,3,6,5,8;
%e PS(3) begins with 1,2,4,6,5,3,7;
%e PS(4) begins with 1,2,4,6,3,7,10.
%o (PARI) get(v, iv) = if (iv > #v, 0, v[iv]);
%o rcp(nbn, nbp, startv, v) = {w = vector(nbn); for (k=1, nbn, if (k % nbp, jv = startv+k, jv = startv+k-nbp); w[k] = get(v, jv);); w;}
%o lista(nn) = {v = vector(nn, n, n); print1(v[1], ", ", v[2], ", "); startv = 3; for (n=3, nn, w = rcp(nn-n+1, n-1, startv, v); startv = 2; if (w[1] == 0, break); print1(w[1], ", "); v = w;);} \\ _Michel Marcus_, Feb 19 2016
%Y Cf. A007062, A057030, A057032, A057033, A057064.
%K nonn
%O 1,2
%A _Clark Kimberling_, Aug 01 2000